K
Khách

Hãy nhập câu hỏi của bạn vào đây, nếu là tài khoản VIP, bạn sẽ được ưu tiên trả lời.

10 tháng 6 2017

Có : với 2 số có tổng không đổi , tích của chúng lớn nhất <=> 2 số đó = nhau(tính chất)(3 số cũng vậy nha :))

=> max P <=> x=y=z=672,(3); nhưng x ; y ; z thuộc N

=> 2 số = 672 ; 1 số = 673

=> max P = 303916032

28 tháng 10 2020

a) Đặt \(\hept{\begin{cases}x+y-z=a\\y+z-x=b\\z+x-y=c\end{cases}\Rightarrow}x=\frac{a+c}{2};y=\frac{b+a}{2};z=\frac{c+b}{2}\)

Suy ra bất đẳng thức cần chứng minh tương đương với: \(\frac{a+b}{2}.\frac{b+c}{2}.\frac{c+a}{2}\ge abc\Leftrightarrow\frac{\left(a+b\right)\left(b+c\right)\left(c+a\right)}{8}\ge abc\)\(\Leftrightarrow\left(a+b\right)\left(b+c\right)\left(c+a\right)\ge8abc\)

Áp dụng bất đẳng thức AM-GM: \(\hept{\begin{cases}a+b\ge2\sqrt{ab}\ge0\\b+c\ge2\sqrt{bc}\ge0\\c+a\ge2\sqrt{ca}\ge0\end{cases}\Rightarrow}\left(a+b\right)\left(b+c\right)\left(c+a\right)\ge8\sqrt{\left(abc\right)^2}=8abc\)

Vật bất đẳng thức được chứng minh

Dấu "=" xảy ra khi \(a=b=c\Leftrightarrow x=y=z\)

7 tháng 9 2021

\(4=x+y+z\ge3\sqrt[3]{xyz}\Leftrightarrow\sqrt[3]{xyz}\le\dfrac{4}{3}\Leftrightarrow xyz\le\dfrac{64}{27}\)(BĐT cauchy)

Dấu \("="\Leftrightarrow x=y=z=\dfrac{4}{3}\)

AH
Akai Haruma
Giáo viên
7 tháng 9 2021

Lời giải:

Áp dụng BĐT AM-GM:
$xy\le \frac{(x+y)^2}{4}=\frac{(4-z)^2}{4}$

$\Rightarrow H\leq \frac{z(4-z)^2}{4}$

Tiếp tục áp dụng BĐT AM-GM:
$z(4-z)\leq \frac{(z+4-z)^2}{4}=4$

$4-z\leq 2$ do $z\geq 2$

$\Rightarrow \frac{z(4-z)^2}{4}\leq \frac{4.2}{4}=2$

Hay $H\leq 2$ 

Vậy $H_{\max}=2$ khi $(x,y,z)=(1,1,2)$

29 tháng 1 2022

Có \(P=\dfrac{x+z}{xyz}=\dfrac{1}{yz}+\dfrac{1}{xy}=\dfrac{1}{y}\left(\dfrac{1}{x}+\dfrac{1}{z}\right)\ge\dfrac{1}{y}.\dfrac{4}{x+z}\)

\(=\dfrac{4}{y\left(x+z\right)}=\dfrac{4}{y\left(4-y\right)}=\dfrac{4}{-y^2+4y}=\dfrac{4}{-\left(y-2\right)^2+4}\ge1\)

"=" xảy ra khi y = 2 ; x = 1 ; z = 1

29 tháng 1 2022

Giúp mình câu này với ah.

 

7 tháng 1 2018

A=x^3 +y^3 +z^3+ 2(x/y+z  +y/z+x  +z/x+y)  \(\ge x^3+y^3+z^3+2.\frac{3}{2}\)  (bạn vào tìm BĐT nesbit là sẽ cm cái đằng sau >= 3/2)

Áp dụng cô si \(x^3+y^3+z^3\ge3xyz=3\)

===> A\(\ge3+3=6\) khi x=y=z=1

4 tháng 10 2017

Bạn tham khảo nhé:

Ta có \(xyz=1\Rightarrow x+y+z\ge3\)

Áp dụng BĐT sờ- swat,ta có:

\(Q\ge\frac{9}{2\left(x+y+z\right)+3}\le1\)(vì \(x+y+z\ge3\))

Vậy max=1

4 tháng 10 2017

Hình như bài này mình bị nghịch dấu rồi

27 tháng 7 2021

Ta có: \(P=\frac{\sqrt{x}}{1+x+xy}+\frac{\sqrt{y}}{1+y+yz}+\frac{\sqrt{z}}{1+z+xz}\)

\(P=\frac{\sqrt{x}}{xy+x+1}+\frac{x\sqrt{y}}{x+xy+xyz}+\frac{xy\sqrt{z}}{xy+xyz+x^2yz}\)

\(P=\frac{\sqrt{x}}{xy+x+1}+\frac{x\sqrt{y}}{xy+x+1}+\frac{\sqrt{xy}.\sqrt{xyz}}{xy+x+1}\)

\(P=\frac{\sqrt{x}+x\sqrt{y}+\sqrt{xy}}{xy+x+1}\le\frac{\frac{x+1}{2}+\frac{x\left(y+1\right)}{2}+\frac{xy+1}{2}}{xy+x+1}\) (bđt cosi)

=> \(P\le\frac{x+1+xy+x+xy+1}{2\left(xy+x+1\right)}=\frac{2\left(xy+x+1\right)}{2\left(xy+x+1\right)}=1\)

Dấu "=" xảy ra<=> x =  y = z = 1

Vậy MaxP = 1 <=> x = y = z = 1